GENERAL PRACTITIONER EXAM
Question Summary
0 of 100 questions completed
Questions:
- 1
- 2
- 3
- 4
- 5
- 6
- 7
- 8
- 9
- 10
- 11
- 12
- 13
- 14
- 15
- 16
- 17
- 18
- 19
- 20
- 21
- 22
- 23
- 24
- 25
- 26
- 27
- 28
- 29
- 30
- 31
- 32
- 33
- 34
- 35
- 36
- 37
- 38
- 39
- 40
- 41
- 42
- 43
- 44
- 45
- 46
- 47
- 48
- 49
- 50
- 51
- 52
- 53
- 54
- 55
- 56
- 57
- 58
- 59
- 60
- 61
- 62
- 63
- 64
- 65
- 66
- 67
- 68
- 69
- 70
- 71
- 72
- 73
- 74
- 75
- 76
- 77
- 78
- 79
- 80
- 81
- 82
- 83
- 84
- 85
- 86
- 87
- 88
- 89
- 90
- 91
- 92
- 93
- 94
- 95
- 96
- 97
- 98
- 99
- 100
Information
Hi, Welcome to General Practioner Exam
You have already completed the Exam before. Hence you can not start it again.
Exam is loading...
You must sign in or sign up to start the Exam.
You have to finish following Exam, to start this Exam:
Results
0 of 100 questions answered correctly
Your time:
Time has elapsed
You have reached 0 of 0 points, (0)
Average score |
|
Your score |
|
Categories
- Not categorized 0%
Pos. | Name | Entered on | Points | Result |
---|---|---|---|---|
Table is loading | ||||
No data available | ||||
- 1
- 2
- 3
- 4
- 5
- 6
- 7
- 8
- 9
- 10
- 11
- 12
- 13
- 14
- 15
- 16
- 17
- 18
- 19
- 20
- 21
- 22
- 23
- 24
- 25
- 26
- 27
- 28
- 29
- 30
- 31
- 32
- 33
- 34
- 35
- 36
- 37
- 38
- 39
- 40
- 41
- 42
- 43
- 44
- 45
- 46
- 47
- 48
- 49
- 50
- 51
- 52
- 53
- 54
- 55
- 56
- 57
- 58
- 59
- 60
- 61
- 62
- 63
- 64
- 65
- 66
- 67
- 68
- 69
- 70
- 71
- 72
- 73
- 74
- 75
- 76
- 77
- 78
- 79
- 80
- 81
- 82
- 83
- 84
- 85
- 86
- 87
- 88
- 89
- 90
- 91
- 92
- 93
- 94
- 95
- 96
- 97
- 98
- 99
- 100
- Unanswered
- Answered
- Review
-
Question 1 of 100
1. Question
1 pointsA 30 year old male has hemoptysis and hematuria. Renal biopsy shows inflammation of the glomeruli. X-ray shows focal pulmonary infiltrates. What is likely to be found with an immunofluorescence study of the glomeruli and alveoli?
Correct
Incorrect
Explanation:
This is a classic case of Goodpasture syndrome, which is autoimmune in origin and characterized by hemorrhagic pneumonitis leading to hemoptysis and glomerulonephritis progressing to renal failure. Most cases involve the presence of anti basement membrane antibodies. It occurs mostly in young men. Under immunofluorescence, there are linear deposits of IgG on alveolar and glomerular basement membranes. Electron dense humps are found with electron microscopy, not immunofluorescence. These humps are found in poststreptococcal glomerulonephritis; immunofluorescence in this case would show C3, IgM, and IgG granular deposits. Mesangial deposition of IgA occurs in Berger disease or IgA nephropathy. Spike and dome IgG deposits are found in membranous nephropathy, a cause of the nephrotic syndrome.
-
Question 2 of 100
2. Question
1 pointsA 63 year old chronic smoker has high blood pressure, moon face, and central obesity. Serum ACTH is increased, but MRI of the pituitary and hypothalamus fail to demonstrate any tumors. CXR reveals a small tumor in the right upper lobe. Histologically, the tumor is composed of sheets of anaplastic cells with a high nuclear/cytoplasmic ratio. The most likely diagnosis is
Correct
Incorrect
Explanation:
Small cell carcinoma of the lung is composed of extremely undifferentiated (i.e., anaplastic) cells, with high nuclear/cytoplasmic ratio. This neoplasm is very aggressive and tends to metastasize so early in its course that even small tumors are considered inoperable by the time of clinical diagnosis. One of the most interesting characteristics of this type of or is the frequent association with paraneoplastic syndromes, the most common of which is due to ectopic production of ACTH. Paraneoplastic syndromes also occur with other types of lung tumors. Adenocarcinomas of the lung are composed of mucin producing neoplastic cells arranged in a glandular pattern. Its histologic features make this tumor easily distinguishable from small cell carcinoma. Carcinoid tumors of the lung are histologically similar to carcinoid tumors of the gastrointestinal tract. They are composed of cords and islands of uniform cuboidal cells. Carcinoid syndrome may be produced because of the ability of the tumor to produce and release serotonin. A hamartoma is not a true neoplasm, but rather a malformative lesion consisting of normal lung tissues arranged in haphazard manner. Hamartomas are often discovered incidentally on chest X-ray films as “coin lesions” and contain cartilage, smooth muscle, and cystlike spaces lined by bronchial epithelium. Squamous cell carcinomas are characterized by neoplastic squamous epithelium with evidence of extracellular and/or intracellular keratin production. The most frequent paraneoplastic syndrome accompanying pulmonary squamous cell carcinomas is hypercalcemia. Finally, remember that cigarette smoking is a strong risk factor for all types of lung cancer, especially squamous cell carcinoma and small cell carcinoma.
-
Question 3 of 100
3. Question
1 pointsTwenty-four passengers set for a Nile cruise. Eighteen of them develop bloody diarrhea on the third day. Which of the following organisms is the likely cause?
Correct
Incorrect
Explanation:
Dysentery is characterised by the passing of frequent (sometimes very frequent) stools that may contain blood, mucus or pus. Shigella dysenteriae is responsible for bacillary dysentery, a disease most often associated with crowded, unsanitary conditions. Other species of Shigella may produce milder forms of diarrhoeal disease. Dysentery is an oral infection transmitted via faecal contamination of water or food. During the one to four day incubation period, penetration of bacteria into the mucosal epithelial cells of the intestine causes an intense irritation of the intestinal wall, Producing cramps and a watery, bloody diarrhoea.
-
Question 4 of 100
4. Question
1 pointsThe change of genetic transmission of Duchene Muscular Dystrophy (DMD) from an affected male who has a child with a normal female, to next generation is
Correct
Incorrect
Explanation:
Duchenne muscular dystrophy is inherited in an X-linked recessive pattern. The following table illustrates the outcomes for an affected male with a normal female and an affected female with a normal male:
X X Xa Xa Xa XaX XaX X XXa XXa Y YX YX Y YXa YXa In first scenario, the affected male mates with a normal female. The female offspring will be carriers and the male offspring will be normal. No offspring will be affected.
In the second scenario, a normal male mates with an affected female resulting in male offspring that are affected and female offspring that are carriers. -
Question 5 of 100
5. Question
1 pointsWhich of the following is false regarding fibroadenoma of the breast?
Correct
Incorrect
Explanation:
Fibroadenomas are typically painless lumps that feel like small, slippery marbles. They usually develop in young women, often in teenagers, and may be mistaken for cancer although they are benign and tend to be more circumscribed and mobile. Simple fibroadenoma does not appear to increase risk of breast cancer; complex fibroadenoma may increase risk slightly. The tumor is not fixed to the adjacent skin, muscle, or lymph nodes, so they are mobile within the breast on palpation. It is commonly found immediately adjacent to the areola, though rarely directly behind the nipple. Macroscopically the tumor is round or ovoid, elastic, nodular, and has a well demarcated capsule; on cut surface it is grey-white. -
Question 6 of 100
6. Question
1 pointsA 35-year-old male presents with severe abdominal pain, nausea, vomiting, and dehydration. On exam, his bowel sounds are diminished and he feels slightly relieved on his left lateral fetal position. He is not jaundiced. On lab data, the lipase level is elevated. Which of the following is NOT associated with his condition?
Correct
Incorrect
Explanation:
This patient has an acute episode of pancreatitis, which can be caused by alcohol, gallstones, blunt abdominal trauma, and medications, such as Lasix, DDI, and chemotherapy and it also has a heritable factor. Pancreatic cancer usually presents as asymptomatic jaundice, weight loss, and depression. The lipase level is not usually elevated in pancreatic cancer.
-
Question 7 of 100
7. Question
1 pointsA 17-year-old female presents with flu-like symptoms that include chills, headache, myalgia, and fever. History reveals a recent tick bite. Which one of the following represents the most likely diagnosis?
Correct
Incorrect
Explanation:
Rocky Mountain spotted fever is caused by Rickettsia Rickettsii. It is transmitted through tick bites (especially Dermacentor andersoni) in the western regions of the U.S.
-
Question 8 of 100
8. Question
1 pointsOut of the following, which ovarian carcinoma is associated with an elevated human chorionic gonadotropin (hCG) level?
Correct
Incorrect
Explanation:
CA-125 may be elevated in ovarian epithelial tumors, such as a cystadenocarcinoma. AFP can be elevated with either an endodermal sinus tumor or embryonal carcinoma. An elevated hCG can be seen with choriocarcinomas. Sertoli-Leydig and granulosa-theca cell tumors do not usually produce an elevated CA- 125, AFP, or hCG level.
-
Question 9 of 100
9. Question
1 pointsTrue statement regarding phenylketonuria is which one of the following?
Correct
Incorrect
Explanation:
A child born with PKU usually appears normal at birth.
-
Question 10 of 100
10. Question
1 pointsSkin condition that is NOT typically associated with diabetes mellitus is
Correct
Incorrect
Explanation:
Porphyria cutanea tarda, a heterogeneous group of disorders of heme biosynthesis, is not a manifestation of diabetes mellitus. All of the other conditions have been described in association with diabetes.
-
Question 11 of 100
11. Question
1 pointsAll of the following are usually associated with goiters, EXCEPT
Correct
Incorrect
Explanation:
Sheehan´s syndrome is also known as postpartum pituitary ischemia and can cause hypothyroidism without the presence of a goiter. All of the other conditions are commonly associated with goiters
-
Question 12 of 100
12. Question
1 pointsWhich one of the following is NOT a mode of transmission of hepatitis C?
Correct
Incorrect
Explanation:
Hepatitis C is not transmitted via well water. The other modes of transmission are all associated with its spread.
-
Question 13 of 100
13. Question
1 pointsWhich of the following statements concerning the thymus is true?
Correct
Incorrect
Explanation:
Cortical thymocytes are immature forms, and either do not express CD4 or CD8 (double negative cells) or express both CD4 and CD8 (double positive cells). As the cells mature, they pass to the thymic medulla where they lose expression of either CD4 or CD8, to become single positive cells.
Negative selection occurs at the stage when thymocytes express both CD4 and CD8, but co-expression of these markers does not mediate negative selection. Negative selection occurs when a thymocyte expresses a TcR with high affinity for self-antigen: MHC complexes in the thymic micro-environment.
Once a thymocyte has successfully rearranged and expressed an -alpha/beta or gamma/delta TcR it is committed to that lineage.
Thymocytes whose TcR bind with high affinity to self Ag/MHC complexes are clonally deleted by a process of negative selection. B cells express IgM and IgD; T cells do not! -
Question 14 of 100
14. Question
1 pointsApoptosis is the process of programmed cell death and occurs in cells that damaged DNA. A mediator of this process is a tumor suppressor gene that inhibits mitosis and promotes apoptosis. This gene is which of the following?
Correct
Incorrect
Explanation:
MYC is a proto-oncogene. Fas is a cell receptor and caspases are present in all cells; both promote apoptosis but are not tumour suppressor genes. Ras is an oncogene.
-
Question 15 of 100
15. Question
1 pointsDeficiency of T-cell response is found in
Correct
Incorrect
Explanation:
DiGeorge syndrome (DGS) consists of congenital thymic aplasia, and thymic hypoplasia.
-
Question 16 of 100
16. Question
1 pointsA 26-year-old male presents with symptoms of gait ataxia and pes cavus. His father developed similar symptoms at the age of 36. Which of the following genetic phenomena explains the age of his presentation?
Correct
Incorrect
Explanation:
A phenomenon whereby the symptoms of a condition appear at an earlier age when inherited in the next generation is termed anticipation. These conditions are almost exclusively associated with trinucleotide repeat sequences and examples are Huntington´s disease, myotonic dystrophy, and Friedrich´s ataxia, as is probably the case in this patient. -
Question 17 of 100
17. Question
1 pointsA 19-year-old attending the A+E department is noted to have central cyanosis. She is perfectly well but was told to go to A+E by her friends who said she looked blue. What is the most likely cause?
Correct
Incorrect
Explanation:
This is typical of methemoglobinemia which may be caused by nitrates. Polycythemia Vera is less likely as it will have more symptoms. Cold and Raynaud´s are causes of peripheral cyanosis. Carbon monoxide is less likely to present like this. -
Question 18 of 100
18. Question
1 pointsA 77 year old man lives alone. He is a smoker and has a typical tea and toast diet. He presents with bleeding gums and loose teeth. The most likely reason is which of the following?
Correct
Incorrect
Explanation:
Vitamin C (ascorbic acid) is essential for the formation, growth, and repair of bone, skin, and connective tissue (which binds other tissues and organs together and includes tendons, ligaments, and blood vessels). Vitamin C helps maintain healthy teeth and gums. It helps the body absorb iron, which is needed to make red blood cells. Vitamin C also helps burns and wounds heal. In adults, vitamin C deficiency usually results from a diet low in vitamin C. For example, vitamin C deficiency may result from diet deficient in fresh fruits and vegetables. Also, cooking can destroy some of the vitamin C in food. Pregnancy, breastfeeding, disorders that cause a high fever or inflammation, surgery, and burns can significantly increase the body´s requirements for vitamin C and the risk of vitamin C deficiency. Smoking increases the vitamin C requirement by 30%. Incidence also peaks in elderly populations, who sometimes have “tea and toast” diets deficient in vitamin C.
-
Question 19 of 100
19. Question
1 pointsA PTT of 42 sec (reference 22 seconds) with a normal PT is NOT suggestive of which one of the following conditions?
Correct
Incorrect
Explanation:
Vitamin K deficiency primarily lengthens the PT and commonly is present with a normal PTT All the other conditions interfere with the PTT-based assessment of clotting but not with the PT-based assessments.
-
Question 20 of 100
20. Question
1 pointsA neoplasm that is NOT considered hereditary is
Correct
Incorrect
Explanation:
Lung cancer is most commonly related to exposure to a carcinogenic agent, such as cigarettes. A hereditary component has not been found. The two multiple endocrine neoplasia syndromes, I and II, are genetically linked and manifest themselves by the occurrence of several specific endocrine tumors in an individual. Retinoblastoma is associated with a specific gene. Familial polyposis coli, with its greatly increased risk of colonic adenocarcinoma, is hereditary.
-
Question 21 of 100
21. Question
1 pointsOvarian carcinoma that is associated with an elevated alpha-fetoprotein (AFP) level is
Correct
Incorrect
Explanation:
CA-125 may be elevated in ovarian epithelial tumors, such as a cystadenocarcinoma. AFP can be elevated with either an endodermal sinus tumor or embryonal carcinoma. An elevated hCG can be seen with choriocarcinomas. Sertoli-Leydig and granulosa-theca cell tumors do not usually produce an elevated CA- 125, AFP, or hCG level;
-
Question 22 of 100
22. Question
1 pointsWhy is the prompt treatment of streptococcal pharyngitis important?
Correct
Incorrect
Explanation:
The most important reason to treat streptococcal pharyngitis promptly and appropriately is to prevent the occurrence of subsequent rheumatic fever, which can result in rheumatic heart disease. Option A is not correct because the clinical symptoms will abate spontaneously without treatment, but the risk of subsequent rheumatic fever remains. Glomerulonephritis and erysipelas are not commonly associated with the strains of streptococci responsible for pharyngitis. Scarlet fever may occur if a patient is not treated, but it is a much less severe complication than rheumatic fever. -
Question 23 of 100
23. Question
1 pointsWhich of the following is NOT a characteristic of adipose tissue deposition?
Correct
Incorrect
Explanation:
Upper body adipocytes are more sensitive to catecholamines and insulin than lower body adipocytes. Fat deposition differs between men and women. The android, male pattern of fat deposition predominantly in the upper body above the waist, is associated with increases in overall morbidity and mortality in both men and women. Upper body fat deposition results primarily from hypertrophy of existing cells, while lower body fat deposition results from an increase in cell number.
-
Question 24 of 100
24. Question
1 pointsObese adults with type II diabetes mellitus often have which of the following?
Correct
Incorrect
Explanation:
So called “adult onset” diabetes in overweight adults is often associated with increased insulin resistance. Such patients are not especially prone to diabetic ketoacidosis or diarrhea. Estrogen receptors have not been implicated in diabetes.
-
Question 25 of 100
25. Question
1 pointsFALSE statement regarding ABO transfusion reactions of the newborn is
Correct
Incorrect
Explanation:
Maternal anti-A and anti-B are primarily IgM and do not cross the placenta. The red blood cells of infants have a decreased number of A, B, and H reactive sites. Therefore, the plasma usually contains very little anti-A or anti-B. Occasionally some type O women produce an IgG cross-reacting antibody, known as anti-A, B, which can cross the placenta and react with fetal cells. When this occurs ABO hemolytic disease results.
-
Question 26 of 100
26. Question
1 pointsAll of the following are manifestations of osteogenesis imperfect, EXCEPT
Correct
Incorrect
Explanation:
Anosmia is not associated with osteogenesis imperfecta, while osteopenia, blue sclerae, collagen mutations, and multiple fractures are all manifestations of osteogenisis imperfect
-
Question 27 of 100
27. Question
1 pointsA 66 year old woman in a nursing facility develops high fever, dyspnea, and a productive cough. The nurses also notice mental status changes, and she is transported to the nearby community hospital. CXR shows a cavitary lesion in her left lung. What organism would likely be identified from sputum examination?
Correct
Incorrect
Explanation:
This woman has developed a pulmonary abscess, as evidenced by the chest radiograph. Of the organisms listed, Staphylococcus aureus is the most likely cause of bacterial pneumonia complicated by abscess formation. Bacteria that commonly produce pneumonia that develops in the context of influenza include S. aureus, Haemophilus influenzae, and Streptococcus pneumoniae but of these S. aureus is most destructive to lung tissue and likely to cause cavitary lesions. S. aureus is a gram positive, catalase positive, coagulase positive coccus. A gram-negative oxidase negative bacillus describes Klebsiella pneumoniae. This bacterium would be a cause of pneumonia in alcoholic or diabetic patients or in those suffering from chronic obstructive pulmonary disease. A gram positive catalase negative, alpha hemolytic diplococcus describes Streptococcus pneumoniae. This agent causes a necrotizing abscessing pneumonia with appearance similar to that of Staphylococcus aureus, but it rarely causes cavitary lesions. A silver-staining cyst describes Pneumocystis jioveci. This agent is associated with atypical pneumonia in the immunocompromised or in premature infants. A yeast that forms pseudohyphae describes Candida albicans. Candida is an opportunistic pathogen in the immunocompromised, but would not be expected to be associated with postviral pneumonia.
-
Question 28 of 100
28. Question
1 pointsA 66 year old man has worsening low back pain, hesitancy and urgency with urination .He denies fever, chills, or any recent sexual contact. A firm, irregular nodule on the posterior surface of the prostate is appreciated on rectal examination. Plasma levels of prostate specific antigen (PSA) are markedly increased. The most likely diagnosis is
Correct
Incorrect
Explanation:
This is the typical clinical presentation of carcinoma of the prostate which is probably the most frequent cancer in males (although lung cancer is the most frequent neoplastic cause of death). Autopsy studies show that small foci of adenocarcinoma are found in the prostate of up to 70% of men older than 80. Prostate-specific antigen (PSA) is expressed by both normal prostatic tissue and adenocarcinoma of the prostate. A rise in PSA plasma levels, therefore, simply results from any increase in the mass of prostatic tissue and cannot be used to differentiate between cancer and hyperplasia. Cancer arises more frequently in the peripheral zone of the gland, producing firm nodules on the posterior surface that can be detected on rectal examination. Metastasis to the bone can be osteoblastic (with formation of new bone) or osteolytic (with destruction of bone). Osteoblastic metastases in the vertebral column are characteristic of prostatic adenocarcinoma. Chronic prostatitis presents with dysuria and perineal discomfort. The diagnosis relies on finding at least 10 leukocytes per high-power field on microscopic examination of expressed prostatic secretions. Chronic prostatitis is further classified into bacterial prostatitis if cultures are positive for bacteria or abacterial prostatitis if cultures are negative. Increased plasma PSA is frequently seen. Multiple myeloma is a neoplasm of plasma cells that frequently manifests with osteolytic (not osteoblastic) lesions in the vertebral column as well as other bones, such as the cranial vault and ribs. Prostatic hyperplasia is an extremely common disorder in middle aged and elderly men. It involves the periurethral zone of the gland leading to compression of the urethra, difficulty in urination, urinary retention, and recurrent cystitis. PSA levels are increased in a manner roughly proportional to the degree of prostatic enlargement. Prostatic intraepithelial neoplasia (PIN) is the precursor of invasive prostatic adenocarcinoma and consists of atypical proliferation of epithelial cells confined within the basement membrane of prostatic acini. It can be found by itself or in prostates with adenocarcinoma. In one third of cases PIN alone progresses to invasive cancer within 10 years. By definition PIN by itself is not associated with metastasis. Other examples of cancer precursors are CIN (cervical intraepithelial neoplasia) and VIN (vulvar intraepithelial neoplasia).
-
Question 29 of 100
29. Question
1 pointsFollowing a severe head injury a 50 year old comatose man was kept on ventilatory support for one week, during which he developed spiking fevers. He died without regaining consciousness. At autopsy lung abscess is seen in the right lung. What condition most likely caused this complication? Correct
Incorrect
Explanation:
This is a severe complication of a number of conditions, prominent among which is aspiration of infective material. Susceptible patients are those with depressed cough reflex, especially comatose patients. In most cases, a mixed bacterial flora, including anaerobic bacteria, are cultured from pulmonary abscesses developing as a result of aspiration. Other causes of lung abscess would be less likely in this patient. Antecedent bacterial pneumonia may complicate into an abscess if the therapy is inadequate or the infective agent is highly virulent. Staphylococcus aureus is the most frequent agent in these cases. Malignant tumors may become secondarily infected and lead to abscess formation. Most commonly, lung cancer manifests with recurrent episodes of bronchopneumonia. Septic embolism may lead to lung abscesses infective endocarditis of the tricuspid valve is the common source of such septic emboli. Spread of infection from contiguous organs is yet another rare cause of lung abscesses. Mediastinitis, esophageal rupture, osteomyelitis of the spine, and subphrenic abscesses are possible causes.
-
Question 30 of 100
30. Question
1 pointsA radiologist notes the presence of line, radiographically dense crystals in the tissues of a knee joint. This patient most likely suffering from which one of the following types of arthropathy?
Correct
Incorrect
Explanation:
Radiographically dense calcium pyrophosphate dihydrate (CPPD) crystals are deposited in cartilage and joint soft tissues in pseudogout, which can involve the knees, wrists, elbows, shoulders, or ankles. The patient is asymptomatic because the deposition is primarily within cartilage, the condition is sometimes called chondrocalcinosis. In its more severe form, the joint involvement can clinically resemble rheumatoid arthritis. Neither gonococcal arthritis, osteoarthritis, nor rheumatoid arthritis is associated with crystal formation.
The monosodium urate crystals deposited in joints in gouty arthritis are not radiopaque. -
Question 31 of 100
31. Question
1 pointsA 1 week old girl with symptoms of vomiting and anorexia has a temperature of 102°F. A bulging fontanel is noted on physical examination. The most likely infectious agent is which one of the following?
Correct
Incorrect
Explanation:
The combination of vomiting, anorexia, high fever (above 100.4°F), and a bulging fontanel equals neonatal meningitis until proven otherwise. Streptococcus agalactiae (group B strep) and Escherichia coli (not an answer choice) are the most common causes in neonates up to 1 month of age. The next most reasonable response would have been Listeria monocytogenes, another, though less common, cause of neonatal meningitis. Most cases of meningitis caused by Haemophilus influenzae occur in children aged 6 months to 6 years; 90% of these cases result from the capsular type b strain. It has become much less prevalent now that the H. influenzae type b conjugate vaccine is routinely administered to infants. Neisseria meningitidis is the most common cause of epidemic meningitis. The two organisms most often associated with sporadic cases are Haemophilus influenzae and Streptococcus pneumoniae (the most common cause in adults older than 30). Staphylococcus aureus is not a common cause of meningitis, except in patients with CSF shunts. It is often responsible for abscesses, osteomyelitis, endocarditis, toxic shock syndrome, and food poisoning.
-
Question 32 of 100
32. Question
1 pointsA 25 year old drug abuser with history of gonorrhea presents with fever and shortness of breath. Pulse oximetry reveals hypoxia. CXR reveals bilateral interstitial infiltrates. The finding that is consistent with the most likely diagnosis is
Correct
Incorrect
Explanation:
is patient has two major risk factors for HIV: drug abuse and a prior sexually transmitted disease (STD), implying high risk behavior. Pneumocystis carinii is the most common cause of pneumonia in HIV positive patients and is diagnosed by demonstration of organisms via methenamine silver stain or fluorescent antibody in lung tissue. Bilateral interstitial infiltrates are characteristic of this disease. Acid fast bacteria found in the sputum is associated with tuberculosis (TB), which is also commonly found in HIV positive patients. Primary TB is usually asymptomatic, whereas reactivation of the infection (secondary TB) is associated with chronic cough, hemoptysis, fever, weight loss, and night sweats. The presence of both a calcified peripheral lung nodule (Ghon complex) and a calcified perihilar lymph node is characteristic of this disease.
Curschmann spirals in mucus plugs, which are shed epithelia that have assumed a spiral configuration, and the presence of Charcot Leyden crystals, which are eosinophils and membrane proteins forming a crystalloid collection, are characteristic of asthma. Elevated serum cold agglutinins (IgMs) are found in 50% of patients with Mycoplasma pneumoniae infection, the most common cause of pneumonia in young adults. Clinical features include fever, malaise, and a dry hacking cough. Twenty percent of patients with adenovirus exhibit this finding as well. -
Question 33 of 100
33. Question
1 pointsA 62 year old hypertensive has severe abdominal pain that radiates to his back. On exam, he is tachycardic, tachypneic, and in acute distress. His BP is 76/40 mm Hg. A large, pulsatile mass can be palpated and an abdominal bruit is auscultated. What is the likely origin of his symptoms?
Correct
Incorrect
Explanation:
This patient is suffering from a ruptured abdominal aortic aneurysm (AAA). Patients typically have abdominal pain that radiates to the back, and AAA should be considered in the differential diagnosis of the acute abdomen. His history of diabetes and hypertension gives him an increased risk for this condition. Rupture of the aneurysm is indicated by this patient´s hemodynamic instability. Neither the descending colon nor the duodenum would likely produce this picture. Severe damage to the inferior vena cava might produce hemodynamic instability, but nothing in the given history indicates this is the case. Stones in the kidney or ureter might produce severe pain, but the pain would typically be in the flank. In addition, renal calculi do not cause the hemodynamic instability seen here.
-
Question 34 of 100
34. Question
1 pointsA 56 year old male has a 3 month history of progressive mental deterioration in the form of memory loss, mood changes, and errors in judgment. His gait is unsteady. No history of seizures, head trauma, or incontinence is present. CT scan and lumbar puncture are unremarkable. Exam reveals hypertonicity of all extremities, bilateral equivocal planter response, ataxic gait, and myoclonic jerks in the lower extremities. The mechanism by which this infectious agent causes its pathology is
Correct
Incorrect
Explanation:
This is the classic presentation of Creutzfeldt-Jacob disease. (CJD).Although the pathogenesis is incompletely understood these patients develop extracellular deposition of abnormal fibrillar proteins in the brain, i.e., amyloid. Autoimmune destruction is not indicated because there is no immunologic response to the deposition of these extracellular proteins; thus there is no chronic inflammation. Although embolization and infarction could complicate the presentation in the age group typically afflicted with CJD these processes are not believed to have any direct role in this pathology. No toxin is produced to account for the presentation in CJD.
-
Question 35 of 100
35. Question
1 pointsA 66 year old female presents with a 30-lb weight loss and malaise. Cancer is suspected. Which is the correct list starting with the most prevalent, of the three most common causes of cancer in woman?
Correct
Incorrect
Explanation:
The correct female incidence sequence is breast (26%), lung (15%), and colorectal (11%). The three major causes of cancer death in women are lung (26%), breast (15%), and colorectal (10%). In men the incidence sequence is prostate (29%), lung (15%), and colorectal (10%). The three leading causes of male cancer deaths are lung (31%), prostate (9%), and colorectal (9%).
The uterus and ovary are not among the three organs most frequently affected by cancer in women. -
Question 36 of 100
36. Question
1 pointsA 2 year old child´s radiograph reveals a new fracture of the humerus and evidence of multiple old fractures in ribs and long bones of the extremities. He shows minimal bruising. Careful examination reveals that he has “peculiar teeth,” a blue tinge to the sclera, and unusually mobile joints. A disease is suspected that is characterized by an abnormality of which biochemical function?
Correct
Incorrect
Explanation:
The child has the most common variant (type I) of osteogenesis imperfecta, which is an autosomal dominant genetic defect in the synthesis of type I collagen, due to decreased synthesis of the procollagen alpha 1 amino acid chain. This defect (unlike that of the perinatal, lethal, type II form of osteogenesis imperfecta) is compatible with survival but does cause skeletal fragility, dentinogenesis imperfecta (abnormal teeth), blue sclera, joint laxity, and hearing impairment. Unfortunately, a number of children with this defect have been removed from their parents because of “abuse,” only to have the broken bones continue in the new environment. Type I collagen is found in skin, bone, tendons, and most other organs. Type II collagen is found in cartilage and vitreous humor. Type III collagen is found in blood vessels, uterus, and skin. Type IV collagen makes basement membranes. Type V collagen is a minor component of interstitial tissues and blood vessels. There are also type VI-XI collagens, which are minor constituents of various tissues.
-
Question 37 of 100
37. Question
1 pointsA 24 year old construction worker falls down on his outstretched right hand. X-ray shows a fracture of one of the wrist bones. The physician is concerned about the risk for avascular necrosis. The wrist bone that was fractured is
Correct
Incorrect
Explanation:
Suspect fracture of the scaphoid (navicular) bone in any young adult who has “fallen on an outstretched hand” (classic clue). The physician was concerned about the risk of avascular necrosis because, in some people, the blood supply of the bone is located distally, and a fracture will deprive the proximal region of the bone of its arterial nourishment. Osteoarthritis is a common complication of scaphoid fractures that do not heal properly. The lunate bone is commonly dislocated in patients who have fallen on an outstretched hand. Median nerve injury is a frequently associated occurrence. The capitate, pisiform, and trapezoid bones of the wrist are less commonly associated with falls on an outstretched hand. -
Question 38 of 100
38. Question
1 pointsA chronic alcoholic male presents with confusion, ataxia, and nystagmus. Administration of thiamine leads to rapid symptomatic improvement. Neuropathologic change that accounts for this presentation is
Correct
Incorrect
Explanation:
The clinical picture of confusion, ataxia, and variable disturbances of eye movements (e.g., nystagmus and ophthahnoplegia) is due to Wernicke encephalopathy, which results from thiamine deficiency. Hemorrhage and necrosis in the above mentioned areas are the underlying pathologic changes. Alcoholics are particularly prone to this condition. Axonal degeneration of posterior and lateral columns of spinal cord, or subacute combined degeneration, results from two different etiologies: vitamin B12 deficiency and HIV myelopathy. Ataxia, numbness, and spastic paresis of the lower extremities are the clinical manifestations. Demyelination affecting the central region of the basis pontis is known as central pontine myelinolysis. This complication is most commonly encountered in severely malnourished and dehydrated alcoholics. Rapid correction of hyponatremia triggers this condition. Demyelination involving the corpus callosum and anterior commissure, or Marchiafava Bignami disease, is a rare disorder associated with chronic alcoholism. It is probably related to some dietary deficiency. Loss of neurons in the neocortex, cerebellar Purkinje cells, and hippocampal pyramidal neurons results from hypoxia or hypoglycemia related necrosis. These neurons are the most vulnerable to hypoxic or hypoglycemic injury. -
Question 39 of 100
39. Question
1 pointsA 32 year old woman presents with fever, night sweats, and weight loss. Examination reveals supraclavicular and lower cervical adenopathy. Lymph node biopsy shows the presence of Reed-Sternberg cells with lacunae surrounding the nuclei of the cells. The variant of Hodgkin disease that is most likely present is
Correct
Incorrect
Explanation:
The nodular sclerosis variant of Hodgkin disease is more common in women and is associated with the presence of lacunar cells [Reed-Sternberg (RS) cells with nuclei surrounded by a clear space] and fibrous bands in the lymph nodes. It has a good prognosis. Lymphocyte depletion is associated with scarcity of lymphocytes, multiple RS cells, fibrosis, and necrosis. It has a poor prognosis. Lymphocyte predominance is the least common form of Hodgkin disease. It is associated with an abundance of lymphocytes and histiocytes and scant RS cells. Mixed cellularity is the most common form of Hodgkin disease. It is associated with the presence of neutrophils, lymphocytes, eosinophils, plasma cells, and histiocytes. Many classic RS cells may be identified.
-
Question 40 of 100
40. Question
1 pointsA child aged 5 years has a 1 month history of fevers and lassitude. He is found to have severe anemia, moderate thrombocytopenia, and a white blood count of 12,000 cells per mm3. What would be likely seen on the bone marrow biopsy?
Correct
Incorrect
Explanation:
Acute lymphoblastic leukemia (ALL) is primarily a disease of children, with peak incidence at 4 years of age. Approximately 80% of child-hood leukemias are of the ALL type. Other features in this scenario that further support an acute leukemia are fevers and anemia without marked elevations of the white blood cell count. Acute myeloblastic leukemia (AML) would present in a similar fashion acute symptoms and anemia with thrombocytopenia. Bone marrow biopsy is needed to definitively differentiate ALL and AML, but AML represents only 20% of childhood leukemias. AML is primarily a disease of adolescents and young adults. The chronic leukemias are diseases of adulthood that present with nonspecific symptoms and are typically diagnosed when white counts are markedly elevated. Chronic myeloid leukemia, a neoplasm of a pluripotent stem cell, also may present with thrombocytosis, rather than thromborytopenia.
Hairy cell leukemia is a relatively rare leukemia of older men. It infiltrates the spleen early in its course and tends to present with pancytopenia due to bone marrow failure and splenic sequestration. -
Question 41 of 100
41. Question
1 pointsA 57 year old smoker and alcoholic, has dysphagia for solids for the 3 months. He also has weight loss and loss of appetite. He had indigestion and heartburn for the past five years. He regularly takes Gaviscon and Rennie tablets. Endoscopy reveals a small tumor at the lower end of the esophagus. The likely etiology for the tumor is which one of the following?
Correct
Incorrect
Explanation:
The history is suggests a five year history of gastro-oesophageal reflux.
This can cause a columnisation of the oesophageal mucosa known as Barrett´s oesophagus, which is a premalignant state. Surveillance endoscopies are recommended every three years. The development of dysphagia for solids and weight loss suggests the presence of oesophageal carcinoma. -
Question 42 of 100
42. Question
1 pointsA homosexual Caucasian HIV positive male aged 50 years develops multiple violaceous painless lesions on his trunk. The most likely cause of his skin lesions is which one of the following?
Correct
Incorrect
Explanation:
Multiple violaceous painless lesions are typical of Kaposi´s sarcoma in Caucasians. It is associated with HHV 8. CMV and HHV 10 do not cause multiple violaceous painless lesions. HPV 16 is an oncogenic virus, which causes squamous cell carcinomas. Pox virus causes molluscum contagiosum
-
Question 43 of 100
43. Question
1 pointsA 26-year-old man presents with proteinuria, haematuria and sensorineural deafness. Which of the following protein structures is likely to be abnormal?
Correct
Incorrect
Explanation:
The diagnosis is Alport´s syndrome, which is a disorder of Type 4 collagen assembly and is inherited as an X-linked (most likely to recessive) disorder in 85% of cases. Fibrillin gene abnormalities are associated with Marfan´s syndrome. Type 1 collagen disorders are associated with osteogenesisimperfecta; it is the main type of collagen in tendon and bone. Type III Collagen is the main component of reticular fibres.
-
Question 44 of 100
44. Question
1 pointsWhich one of the following is a recognized feature of achondroplasia?
Correct
Incorrect
Explanation:
Achondroplasia, an autosomal dominant condition and one of the commonest forms of inherited dwarfism. Epiphyseal dysplasia- thin zone of cartilage cells diminished columnar arrangement short thick bone, spinal length almost always normal. Features- short limbs, normal trunk , large head, saddle nose, exaggerated lumbar lordosis normal mental and sexual development, spinal problems. Homozygotes- neonatal death.
-
Question 45 of 100
45. Question
1 pointsA 24 year old woman develops generalized pruritus and hyperemia of the skin, followed shortly by swelling of the face and eyelids, dyspnea, and laryngeal edema a few minutes after eating peanuts. This reaction is mediated by?
Correct
Incorrect
Explanation:
An allergic reaction is when the immune system mistakes a harmless substance for dangerous one and produces an antibody called “immunoglobulin E”, more commonly known as “IgE”. When an allergic person is exposed to an allergen, IgE is produced to fight off the specific allergen. For example, if someone is allergic to peanuts and eats some, then an “anti-peanut” IgE antibody is produced. Each time IgE is produced, the IgE molecules attach themselves to mast cells that are found in large numbers in the eyes, nose, lungs, intestines, and immediately beneath the skin. These mast cells contain many chemicals, including a substance called histamine which, when released into the body, can cause runny nose, sneezing, watery eyes, itching, hives, and wheezing. These effects are recognized as allergy symptoms. In some cases, reactions can occur in several places throughout the body. Welts or hives may appear, spasm in the lungs may cause coughing or wheezing, the throat and tongue may swell; if anaphylaxis (severe allergic reaction) occurs, it may be fatal. -
Question 46 of 100
46. Question
1 pointsWhich one of the following viruses is most likely to cause a chronic hepatitis infection?
Correct
Incorrect
Explanation:
Hepatitis C is the most likely to progress to a chronic carrier state (approximately 50% of cases). Hepatitis B and D also may cause a chronic infection. Hepatitis A and E do not cause chronic liver infection.
-
Question 47 of 100
47. Question
1 pointsWhich one of the following is most diagnostic of meningitis in patients with cancer?
Correct
Incorrect
Explanation:
Listeria monocytogenes and cryptoccus spp. are important in the onset of neoplastic meningitis. Listeria monocytogenes is an intracellular bacterium, which evades the immune system. When it causes meningitis in cancer patients, it can be a cause of significant morbidity and even death.
-
Question 48 of 100
48. Question
1 pointsWhich of the following promotes edema formation?
Correct
Incorrect
Explanation:
Derangements of Starling forces explain edema formation. Increased capillary hydrostatic pressure, decreases in capillary oncotic pressure, or a combination of the two results in edema formation. Damage to the capillary endothelium increases permeability permitting transudation of protein and fluids into the interstitium. Elevated interstitial hydrostatic pressures tend to move fluid into the vasculature space.
-
Question 49 of 100
49. Question
1 pointsWhich of the following is NOT a metabolic alteration in severe acute hypoxia?
Correct
Incorrect
Explanation:
Lactic acid cannot be further metabolized, resulting in lactic acidosis. Normally in liver and muscle, metabolism of carbohydrates, the primary foodstuff, proceeds anaerobically up to the stage of pyruvate formation. Degradation of pyruvate requires oxygen. In hypoxic states, increasing proportions of pyruvate are degraded to lactic acid. Metabolic acidosis occurs, resulting in decreased total energy output from the breakdown of carbohydrates. The amount of energy available for continuing resynthesis of energy-rich phosphate compounds becomes inadequate.
-
Question 50 of 100
50. Question
1 pointsA patient with uncontrolled diabetes mellitus and acidosis probably has which one of the following?
Correct
Incorrect
Explanation:
A patient with uncontrolled diabetes mellitus and acidosis probably has ketoacidosis. Lactic acidosis is most common in individuals with severe circulatory abnormalities in the absence of other forms of acidosis. Mixed metabolic acidosis and respiratory alkalosis do not present in the setting of uncontrolled diabetes and acidosis. Uncontrolled diabetes causes an anion gap due to ketone bodies, thus making normal anion gap acidosis an incorrect answer. Mixed metabolic acidosis and respiratory acidosis are not associated with uncontrolled diabetes.
-
Question 51 of 100
51. Question
1 pointsGlomerulonephritis (GN) that is associated with streptococcus is which of the following?
Correct
Incorrect
Explanation:
Berger`s disease is a mesangial IgA deposition causing hematuria, but no decline in renal function. Henoch Schonlein purpura is characterized by a nephritis, blood diarrhea, abdominal pain, arthritis, and purpura. Goodpasture´s syndrome causes an acute GN and pulmonary hemorrhage by producing an antiglomerular basement membrane antibody. Streptococcal infections can produce postinfectious glomerulonephritis. Alport´s syndrome is hereditary hematuria associated with sensorineural hearing loss and eye abnormalities.
-
Question 52 of 100
52. Question
1 pointsPolycystic kidney disease is NOT associated with which of the following
Correct
Incorrect
Explanation:
Bronchial cysts are not associated with polycystic kidney disease. Patients with polycystic kidney disease can have associated hepatic cysts. They also are at risk for chronic renal insufficiency, hypertension, and renal failure, due to progressive renal damage.
-
Question 53 of 100
53. Question
1 points“Gout” is best matched with which one of the following answer?
Correct
Incorrect
Explanation:
Gout is associated with monosodium urate crystals.
-
Question 54 of 100
54. Question
1 pointswhich one of the following is NOT a cause of female infertility?
Correct
Incorrect
Explanation:
Bromocriptine is used to treat hyperprolactinemic individuals with anovulation. Female factors account for approximately half of the cases of infertility. In luteal phase dysfunction, there is insufficient progesterone formed to permit endometrial implantation. Infertility caused by implantation of endometrial tissue may require surgical resection or treatment with gonadotropin suppression. Abnormalities of the uterine wall, such as leiomyomas or polyps, may also lead to infertility. Immunologic factors, although rare, such as sperm immobilizing or agglutinating antibodies, may also result in infertility.
-
Question 55 of 100
55. Question
1 pointsFALSE statement regarding clostridial myonecrosis is which one of the following?
Correct
Incorrect
Explanation:
Beta toxin is thought to increase capillary permeability. Clostridial myonecrosis (gas gangrene) typically develops following deep, penetrating traumatic injury. The initiating trauma introduces vegetative forms or spores into the deep tissues. The low oxidation reduction potential and acidic pH are optimal for bacterial growth. The bacterium elaborates many toxins. At the site of infection, theta toxin, in high concentrations, destroys host tissues and inflammatory cells. Alpha toxin is a lecithinase and is known to be hemolytic. It may contribute to hypotension by suppressing myocardial contractility thereby reducing cardiac output.
-
Question 56 of 100
56. Question
1 pointsA 21 year student presents with dysmenorrhea and lower abdominal tenderness. Pelvic examination shows adnexal tenderness and mucopurulent discharge without bleeding. A scraping from the cervix is stained with Giemsa stain shows intracytoplasmic inclusion bodies in the infected epithelial cells. Unusual statement about the biochemical makeup of this organism is
Correct
Incorrect
Explanation:
This young woman is infected with Chlamydia trachomatis. This genus of organisms is unusual in that they do not make ATP and are thus obligate intracellular parasites, and they possess a cell wall in which muramic acid is missing from the peptidoglycan. Thus they have a cell wall, but it is made of highly modified peptidoglycan (it can be called peptidoglycan at all). It has sterols in its membrane is a statement that would describe the mycoplasmas. These are the only prokaryotes that totally lack a cell wall, but instead have sterols in their membranes. It lacks a cell wall would be a statement that describes the mycoplasma. These prokaryotes are devoid of a cell wall, but have sterols in their membranes. Its cell membrane contains ergosterol would describe the fungi. This molecule is unique to fungi and is the target molecule for nystatin and the imidazoles. A cell wall containing mycolic acids describes the genus Mycobacterium. Mycolic acids are long chain fatty acids that give this group its resistance to environmental stresses and its acid fast staining characteristics.
-
Question 57 of 100
57. Question
1 pointsA 21 year old college student working part time in a pediatric clinic develops a viral exanthem with a rash. Her blood is tested for specific antibodies to varicella zoster. Which antibody class´s antivaricella immunoglobulin would indicate that she is immune to chickenpox?
Correct
Incorrect
Explanation:
The immunoglobulin that represents past exposure to an antigen is IgG. It is the predominant antibody in the secondary response and is an important defense against bacteria and viruses. In the secondary response, a much larger amount of IgG antibody is produced than IgM, and the levels of IgG tend to persist for a much longer time than in the primary response. IgG is an opsonic antibody: It binds to Fc receptors on phagocytic cells to enhance the speed of phagocytosis. An antibody with two epsilon chains describes IgE. It is produced during atopic allergies but would not be associated with immunity to a viral pathogen. An antibody with a valence of 4 describes IgA. The valence of an antibody is the number of combining sites per molecule, and since IgA is a dimer, it has a total of four combining sites. An antibody with a valence of 10 describes IgM. IgM is a pentameric molecule with a total often combining sites. The antibody with the shortest half life describes IgD. This serves as the membrane receptor on the surface of mature naive B lymphocytes along with IgM. It has no known biologic functions other than as a membrane receptor.
-
Question 58 of 100
58. Question
1 pointsA 40 year old woman is brought with severe abdominal pain localized to the right upper quadrant. A urine sample is taken for rapid dipstick analysis. A positive result for which substance would strongly suggest gallstone disease as a possible cause of her abdominal pain?
Correct
Incorrect
Explanation:
A small gallstone passing into the common bile duct can cause obstructive jaundice with conjugated hyperbilirubinemia. The conjugated bilirubin will spill into the urine, causing the bilirubin square on the reagent strip to react. This strip may also react with other causes of intrahepatic or extrahepatic obstructive jaundice, so it is not completely specific. It is worth becoming very familiar with the strengths and weaknesses of reagent strip technology, however, since this information may be available hours before serum chemistry values are reported. High glucose in urine suggests diabetes mellitus.
High nitrite in urine suggests urinary tract infection. Unconjugated bilirubin is not filtered by the glomerulus because it is water insoluble; therefore, it cannot be identified on urine dipstick. Urobilinogen levels in obstructive jaundice can be normal, raised, or lowered; consequently, they are not diagnostically helpful in the setting described in this question stem unless urinary bilirubin is negative. They are typically elevated in hemolytic conditions. -
Question 59 of 100
59. Question
1 pointsTesticular biopsy of a 20 year old boy with persistent gynecomastia shows fibrosis and hyalinization of the seminiferous tubules. The Leydig cells are present in clumps and are hyperplastic. Presence of an XXY genotype is seen on chromosomal analysis. Which lab finding would be most likely in this individual?
Correct
Incorrect
Explanation:
Klinefelter syndrome is characterized by an XXY genotype and is associated with seminiferous tubule dysgenesis. In this disorder there are usually few symptoms before puberty, which may be delayed. At puberty, the seminiferous tubules fail to enlarge normally and instead undergo fibrosis and hyalinization. The result is an inability to produce sperm. The Leydig cells are hyperplastic and clumped together, and do not function normally (secretion of testosterone is decreased). As a consequence, plasma levels of LH are increased (not decreased) due to the loss of feedback inhibition. The increased LH stimulates the Leydig cells to increase (not decrease) estrogen secretion. The increased estrogen/testosterone ratio is responsible for the gynecomastia, small penis, sparse body hair, and other feminized features of Klinefelter syndrome. Because of dysgenesis of the seminiferous tubules the secretion of inhibin is reduced (not increased). Low plasma levels of inhibin result in increased plasma FSH (not decreased) due to loss of feedback inhibition at the pituitary.
-
Question 60 of 100
60. Question
1 pointsA 50 year old smoker presents with fever and a cough productive of greenish-yellow sputum. He has had a morning cough with excessive mucus production for the past 5 years. The abnormality that would be found in him is
Correct
Incorrect
Explanation:
This patient presents with symptoms suggestive of acute infection (elevated temperature, greenish-yellow sputum) on a background of chronic bronchitis which is common in smokers. Hyperplasia and hypertrophy of mucous glands in chronic: bronchitis causes them to be present at deeper levels in the bronchial wall than usual. The ratio of the gland depth to the, total thickness of the bronchial wall is termed the Reid index, which would be increased in this patient. Apical cavitary lesions might be indicative of cavitary tuberculosis. This condition is not associated with excessive mucus production. Hemoptysis and weight loss might also be expected as clinical findings. Curschmann spirals are found in asthmatic patients and represent mucus casts of small airways. This patient does not have the typical episodic history of acute asthmatic attacks with acute dyspnea as the major clinical problem. Elevated sodium chloride levels in sweat are present in cystic fibrosis. This condition has n onset in early live and is associated with excessive production of thick mucus, which predisposes to infection of the airways and permanent damage. Enlarged hilar lymph nodes might suggest bronchogenic carcinoma or a granulomatous process which would be less likely than chronic bronchitis. In addition, patients with carcinoma often present with hemoptysis and weight loss, rather than excessive mucus production.
-
Question 61 of 100
61. Question
1 pointsA 36-year-old woman presents with pains in the left arm. On examination she has wasting and weakness of the intrinsic muscles of the left hand, absent tendon reflexes in the left arm and impaired pinprick sensation in the left hand and forearm. What is the most likely diagnosis?
Correct
Incorrect
Explanation:
Median and ulnar nerve lesions would not cause absent reflexes in the arm. Lower trunk brachial plexus (C8/T1) would not cause absent reflexes in the arm. Neuralgic amyotrophy affects the upper plexus (C5-6) and therefore does not cause wasting of small muscles of hand. Thoracic outlet syndrome will not cause absent reflexes. Syringomyelia typically causes loss of reflexes, spinothalamic sensory loss, and weakness. It can be asymmetrical initially. -
Question 62 of 100
62. Question
1 pointsDamage to the gastric mucosa by Helicobacter pylori is facilitated by all of the following EXCEPT
Correct
Incorrect
Explanation:
Monocytes are activated to express HLA-DR and interleukin 2 receptors on their cell surfaces and to produce superoxides, interleukin I, and tumor necrosis factor alpha. Helicobacter pylori produce a variety of proteins which facilitate its damaging effects on the gastric mucosa. Urease catalyzes the hydrolysis of urea to form ammonia and carbon dioxide. The resulting alkaline environment protects the organism from gastric acid and prevents gastric colonization by other bacteria. Surface proteins are chemotactic for neutrophils and monocytes. Proteases and phospholipases degrade the glycoprotein-lipid complex of the mucus gel layer, thereby reducing the thickness and viscosity of this protective layer. Adhesin facilitates the attachment of the organism to gastric epithelial cells.
-
Question 63 of 100
63. Question
1 pointsA researcher is studying ischemic myocardial necrosis using an animal model. Myocardial damage is evaluated histologically at different times following irreversible ischemic injury produced by ligation of a coronary branch. Necrotic changes would first be visible by light microscopy at which of the following times?
Correct
Incorrect
Explanation:
Irreversible ischemic injury to the myocardium occurs following interruption of blood supply that lasts at least 20-30 minutes. However, necrotic changes only become apparent on light microscopy 4 to 12 hours after ischemia. This implies that the myocardium of a person who is dying, for example, 20 minutes or 3 hours after suffering a myocardial infarction will appear entirely normal on gross and microscopic examination. Electron microscopic studies reveal morphologic changes at earlier stages demonstrating sarcolemmal disruption and mitochondrial swelling following irreversible ischemia. This fact has important implications of a clinical and medico-legal nature. Note that the cellular changes of necrosis do not occur after death. Necrosis is a biologic phenomenon that, paradoxically, may only develop in a living organism. After 3 days, coagulative necrosis will be advanced. Myocytes will have lost their nuclei and the cytoplasm will be disintegrating. Typically, there is abundant neutrophilic infiltration admixed with histiocytes. This influx of inflammatory cells is necessary to remove necrotic debris prior to the development of granulation tissue, which is far advanced at about 5 to 7 days after ischemia.
Time Following Coronary Occlusion Microscopic Findings 20 minutes None 3 hours None 10-12 hours Initial necrotic changes 3 days Advanced coagulative necrosis with inflammatory cells 5-7 days Development of granulation tissue -
Question 64 of 100
64. Question
1 pointsGlomerulonephritis (GN) that is associated with streptococcus is which of the following?
Correct
Incorrect
Explanation:
Berger`s disease is a mesangial IgA deposition causing hematuria, but no decline in renal function. Henoch Schonlein purpura is characterized by a nephritis, blood diarrhea, abdominal pain, arthritis, and purpura. Goodpasture´s syndrome causes an acute GN and pulmonary hemorrhage by producing an antiglomerular basement membrane antibody. Streptococcal infections can produce postinfectious glomerulonephritis. Alport´s syndrome is hereditary hematuria associated with sensorineural hearing loss and eye abnormalities
-
Question 65 of 100
65. Question
1 pointsWhich of the following is NOT a characteristic of adipose tissue deposition?
Correct
Incorrect
Explanation:
Upper body adipocytes are more sensitive to catecholamines and insulin than lower body adipocytes. Fat deposition differs between men and women. The android, male pattern of fat deposition predominantly in the upper body above the waist, is associated with increases in overall morbidity and mortality in both men and women. Upper body fat deposition results primarily from hypertrophy of existing cells, while lower body fat deposition results from an increase in cell number.
-
Question 66 of 100
66. Question
1 pointsThe organism LEAST likely to be transmitted by sexual activity is which one of the following?
Correct
Incorrect
Explanation:
Shigella is uncommonly transmitted by sexual intercourse. All of the other conditions are primarily transmitted by sexual activity
-
Question 67 of 100
67. Question
1 pointsAll of the following are usually associated with goiters, EXCEPT
Correct
Incorrect
Explanation:
Sheehan´s syndrome is also known as postpartum pituitary ischemia and can cause hypothyroidism without the presence of a goiter. All of the other conditions are commonly associated with goiters.
-
Question 68 of 100
68. Question
1 pointsIngestion of which one of the following agent would most likely increase the risk of breast cancer in women with strong family histories of breast cancer?
Correct
Incorrect
Explanation:
An increased incidence of breast cancer, especially in women with strong family histories, has been reported with the ingestion of exogenous estrogen.
-
Question 69 of 100
69. Question
1 pointsA colony of Neisseria gonorrhoeae is isolated on Thayer Martin medium. Over the course often generations in culture, this strain produces a homogeneous antigenic type of pili. Lysates from a culture of N. gonorrhoeae with a different pilus type are added to the stable colonies. The bacteria are cloned after overnight incubation and their pilus expression is analyzed. A new antigenic form of pilus is new expressed in 50% of the new colonies. These cultures have gone through which one of the following?
Correct
Incorrect
Explanation:
When lysates of bacteria are added to cultures, the DNA can be taken up by the living cells by the process of transformation. Many bacteria are capable of transformation following laboratory treatments that allow them to endocytose extracellular DNA. Only a few medically important species undergo natural transformation: Haemophilus species, Streptococcus species, Neisseria gonorrhoeae, and Helicobacter pylori. These organisms are capable of taking up free DNA without pretreatment. Other bacteria may be induced to undergo artificial transformation in the laboratory. In N. gonorrhoeae, it is estimated that up to a million antigenically distinct types of pili can be formed by recombination of expressed and silent DNA sequences within a cell, as well as by uptake of DNA from other cells in the environment. Because the pili are structures on the outside of the bacterium changing their antigenic composition during an infection gives the organism an advantage against the host´s immune response. Conjugation is a mechanism of genetic exchange in bacteria that involves cell to cell contact and the donation of DNA from a donor cell to a recipient. Because this experiment stipulates that the lysates of cells are added to the cultures, there can be no cell to cell contact. Generalized transduction is the mechanism of genetic exchange in bacteria that involves the presence of a virus vector. In this manner, virulent phage can accidentally transfer DNA from one bacterium to another. There is no mention of the presence of viruses in this question stem. Site specific recombination is the mechanism by which two circular places of DNA can be combined with one another. It occurs during the process of specialized transduction during the life cycle of lysogenic viruses, during transposon movement, and is the means by which the Hfr chromosome is formed. It´s not a means by which extracellular linear places of DNA are imported into cells, as described here. Specialized transduction is the mechanism of genetic exchange in bacteria that involves transfer of bacterial genes from cell to cell using a lysogenic virus. There is no mention of viruses in the question stem here.
-
Question 70 of 100
70. Question
1 pointsA 46 year old female presents with finger stiffness. Examination shows marked thickening of the skin, most striking on the hand which is limiting finger mobility and multiple small telangiectasias and several hard nodules on the buttocks. She has recent difficulty swallowing and skin color changes when the hands are exposed to cold. Autoantibody formation against which substance is most strongly associated with her condition?
Correct
Incorrect
Explanation:
The patient has the CREST variant of scleroderma, which is associated with anticentromere antibody. The CREST syndrome comprises calcinosis (the hard calcified, subcutaneous nodules of the buttocks), Raynaud phenomenon (the skin color changes in response to cold), esophageal dysmotility (difficulty swallowing), sclerodactyly (scleroderma involving the fingers), and telangiectasia (small vascular lesions of the skin). The CREST variant usually has a more benign course than the systemic variant of scleroderma. Anti-double stranded DNAand anti-Smith antigen are associated most strongly with systemic lupus erythematosus. Anti-glomerular basement membrane is associated with Goodpasture disease. Anti-Sci 70 is associated with the systemic form of scleroderma.
-
Question 71 of 100
71. Question
1 pointsIn studies of mice that were experimentally infected with leishmania, two inbred strains are found to have widely divergent responses to the pathogen. In one strain, the C57BL/6 mouse, amastigotes are effectively killed intracellularly. In the other strain, the BALB/c mouse, proliferation of the amastigotes inside phagocytic cells proceeds unchecked, and the animals fail to resist the infection. The cytokine that is likely to be responsible for the resistance of the C57BL/6 mouse is
Correct
Incorrect
Explanation:
Because this is a protozoan that survives inside macrophages, the most effective immune response is the cell-mediated one and stimulation of macrophage-killing mechanisms. The most important cytokines to stimulate macrophage microbicidal activity are those produced by TH1 cells, namely interferon-gamma (IFNγ). Interleukin-2 is a product of TH cells that causes the proliferation (cloning) of other lymphocytes. Because the macrophage is an end cell, incapable of further replication, it would g not be affected by IL-2 administration. Interleukin-4 is a product of TH2 cells that causes the differentiation of B lymphocytes. It inhibits TH1 cells that would assist in this protective response. This is a cytokine that is likely to be elevated in the extremely susceptible BALB/c mouse. Interleukin-10 is a product of TH2, cells that causes the differentiation of B lymphocytes. It inhibits TH1 cells, which would assist in this protective response. This is a cytokine that is likely to be elevated in the extremely susceptible BALB/c mouse. Tumor necrosis factor-alpha is a product of macrophages and NK cells that is cytotoxic for tumor cells, induces cytokine production and causes the cachexia of chronic inflammation. It is unlikely to have a beneficial result on the microbicidal activity of macrophages. -
Question 72 of 100
72. Question
1 pointsA 12 year old girl brought for a scheduled inguinal hernia repair is curious about her menarche because most of her friends already have their period. She has breast buds, a vaginal pouch, and no uterus, and no axillary or public hair. Karyotype analysis shows 46, XY. Labs show high testosterone levels. These clinical and lab findings are likely the result of which mechanism?
Correct
Incorrect
Explanation:
This patient most likely has testicular feminization, which is androgen insensitivity due to an androgen receptor defect. These individuals are phenotypically female and may present with an inguinal hernia or primary amenorrhea. The clinical features include female external genitalia, a vaginal pouch, no uterus, feminized breasts and a paucity of axillary and pubic hair. Laboratory studies show a 46, XY karyotype and male levels of serum testosterone. These patients have a truly feminine appearance. Undescended testes are present and should be removed, usually alter puberty. Estrogen replacement therapy is usually indicated. Congenital adrenal hyperplasia (CAH) is typically caused by deficiencies of 11-beta-hydroxylase or 21-hydroxylase and is inherited as an autosomal recessive trait. Affected females who have female chromosomes have increased androgen production that leads to virilization. They have a vagina and uterus, but there is hypertrophy of the clitoris with ventral binding and fusion of the labioscrotal folds. Treatment includes the surgical correction of the external genitalia and glucocorticoids. The patient in this case is a genotypic male and a phenotypic female, which is inconsistent with CAH. Excess adrenal androgen resulting in extraglandular estrone production is likely the mechanism for polycystic ovarian disease which is characterized by anovulation, amenorrhea or oligomenorrhea, obesity, and hirsutism. There is an elevated LH level. These patients are genotypic females, unlike the patient in this case. Individuals with an imperforate hymen, which is caused by incomplete genital plate canalization typically present with primary amenorrhea, cyclic abdominal pain, and a bulging introitus. Menarche occurs at the appropriate time in these females, however the blood flow is obstructed. The treatment is a hymenotomy. -
Question 73 of 100
73. Question
1 pointsAnalysis of a man with excessive thirst and urination reveals serum osmolarity of 310 Osm/L and urine osmolarity of 90 mOsm/L. Plasma glucose is normal. Water deprivation fails to increase urine osmolarity. Vasopressin injection also fails to increase urine osmolarity. What is the most likely diagnosis?
Correct
Incorrect
Explanation:
Nephrogenic diabetes insipidus results in excessive excretion of free water in the urine because of an inability of the kidney to respond to vasopressin (antidiuretic hormone). It can occur in association with certain renal diseases that prevent the normal formation of the medullary concentration gradient. In this case, the kidney is unable to produce concentrated urine. Congenital nephrogenic diabetes insipidus could be due to a defect in the renal V2 receptor, Gs protein, or other steps in the normal formation of cyclic AMP. Plasma levels of vasopressin are usually increased because of the hyperosmolarity of the serum. Water deprivation will fail to increase urine osmolarity in both neurogenic and nephrogenic diabetes insipidus. However, subsequent injection of vasopressin will concentrate the urine in the case of neurogenic diabetes insipidus, but not in nephrogenic diabetes insipidus. Certain drugs (e.g. lithium) can also produce similar symptoms. Diabetes mellitus causes polyuria because of an osmotic diuresis due to glucosuria. This sort of diuresis does not involve the loss of much free water and the urine osmolarity lends toward that of the plasma (not 90 mOsm/L, as in the patient above). Furthermore, plasma glucose is normal in this patient, making diabetes mellitus unlikely. Some patients with primary hyperparathyroidism complain of increased urination. This is due to an osmotic diuresis, produced in this case by hypercalciuria. When serum levels of calcium exceed 12 mg/dL, the kidney´s ability to reabsorb filtered calcium is overwhelmed and hypercalciuria ensues. Primary polydipsia is a psychological disorder characterized by excessive water drinking. It too produces polyuria, but the excretion of free water is appropriate. These patients typically present with decreased serum osmolarity (due to the dilutional effect of the ingested water). Furthermore, water deprivation should produce concentrated urine. It is helpful to remember that the water deprivation test can distinguish between primary polydipsia and diabetes insipidus, but it cannot distinguish between neurogenic and nephrogenic diabetes insipidus. -
Question 74 of 100
74. Question
1 pointsA 21 year old female has a 2 day history of dysuria and increased urinary frequency. She was recently married and was not sexually active before marriage. Exam reveals a temperature of 100.7°F with normal vital signs. No evidence of discharge, vaginitis, or cervicitis is revealed. Urinalysis reveals 14 WBCs per high-powered field with many gram negative rods. What is the appropriate treatment?
Correct
Incorrect
Explanation:
The patient´s presentation is consistent with a simple urinary tract infection; there is a short history of dysuria, increased urinary frequency and the appearance of white blood cells and gram negative rods in the urine. Urinary tract infections are common in women after they become sexually active. The infection is likely caused by urethral trauma during intercourse, which leads to bacterial contamination of the bladder. Since most of these infections are caused by Escherichia coli (a gram-negative rod), the most appropriate therapy would be ampicillin for around 10 days. Ceftriaxone is the treatment of choice for uncomplicated infections with Neisseria gonorrhoeae, now that most strains are resistant to penicillin. IV ceftriaxone is a regimen reserved for the treatment of life threatening infections. Fluconazole is indicated for the treatment of vaginal candidiasis. Since there is no vaginal discharge and the patient has gram-negative rods in the urine, a diagnosis of vaginal candidiasis can be excluded. Gentamicin would be an inappropriate choice. Most urinary tract infections caused by gram negative rods are sensitive to ampicillin, and the potential for toxicity secondary to gentamicin is great. Metronidazole is an antibiotic typically used in the treatment of vaginal Trichomonas and Gardnerella infections, as well as serious infections believed to be caused by anaerobic bacteria. Since there is no vaginal discharge and the patient has gram negative rods in the urine, this is not the best choice for treatment.
-
Question 75 of 100
75. Question
1 points16 year old boy develops diarrhea and stomach ache after eating a dinner of leftovers that included rewarmed vegetable fried rice. The most likely causative organism is which one of the following?
Correct
Incorrect
Explanation:
Bacillus cereus contaminates grains, such as rice, and produces spores resistant to quick frying and steaming. The other most important clue for this etiology would be an onset of symptoms 1-2 hours after eating. Campylobacter jejuni causes enterocolitis with bloody diarrhea, crampy abdominal pain, malaise, and fever. Clostridium botulinum produces a constellation of signs and symptoms, including bulbar palsy, descending weakness or paralysis, progressive respiratory weakness, absence of fever, dry mucous membranes, and autonomic dysfunction. Clostridium difficile causes pseudomembranous colitis, classically resulting from clindamycin use. Escherichia coli comes in a variety of forms. The enterotoxigenic type is the most common cause of traveler´s diarrhea.
-
Question 76 of 100
76. Question
1 pointsA sexually active 35 year old male presents with a painless penile vesicle. Exam reveals inguinal lymphadenopathy. The organism is definitively diagnosed and is known to exist in distinct extracellular and intracellular forms. The most likely infectious agent is which one of the following?
Correct
Incorrect
Explanation:
This patient has lymphogranuloma venereum caused by Chlamydia trachomatis (type L1, 2, or 3). Chlamydia exhibit distinct infectious and reproductive forms. The extracellular infectious form is known as the elementary body (EB), which cannot reproduce. It attaches to the host cell and enters through endocytosis. Once inside the cell, the EB is transformed into the reticulate body (RB) within the endosome. The RB is capable of binary fission and divides within the endosome; fusion with other endosomes occurs to form a single large inclusion. Eventually, the RBs undergo DNA condensation and disulfide bond bridgings of the major outer membrane protein, forming EBs. The EBs are then released. Note that C. trachomatis is responsible for several sexually or perinatally transmitted diseases, including ocular trachoma (types A, B, and C), neonatal conjunctivitis, nongonococcal urethritis, cervicitis, and pelvic inflammatory disease (types D-K).
Calymmatobacterium granulomatis is a gram-negative rod that causes superficially ulcerated genital or inguinal papules that coalesce to form substantial lesions. The appearance of Donovan bodies in histiocytes is diagnostic of this infection. Haemophilus ducreyi is a gram-negative rod that causes a soft, painful penile chancre, unlike that of a chlamydial or syphilitic lesion. This infection is common in the tropics. Neisseria gonorrhoeae is a gram-negative diplococcus responsible for gonorrhea. Patients typically present with purulent penile discharge, not genital lesions. Treponema pallidum is the spirochete responsible for syphilis. It may cause a firm, painless ulcer as a manifestation of primary syphilis, but the organism does not exist in distinct extracellular and intracellular forms as does Chlamydia. Secondary syphilis is associated with the appearance of condylomata lata-flat, gray, wart like lesions. -
Question 77 of 100
77. Question
1 pointsA boy aged 13 years presents with a deep skin abrasion on his knee. It has not stopped bleeding since it happened during recess approximately 20-30 minutes ago. Exam reveals a healthy adolescent. There are multiple purpura over his legs and arms, and a few scattered petechiae on his chest and gums. His bleeding time = 22 minutes, platelets = 300,000/mm3, and Hb= 11g/dL. Trial of cryoprecipitate transfusion does not improve his bleeding time, but a normal platelet transfusion does. The most likely diagnosis is which one of the following?
Correct
Incorrect
Explanation:
Bernard Soulier syndrome is an autosomal recessive disease of platelet adhesion that causes prolonged bleeding times in the presence of normal platelet counts. These patients´ platelets cannot bind to subendothelial collagen properly because of a deficiency or dysfunction of the glycoprotein Ib-IX complex. Clinically the patients have impaired hemostasis and recurrent severe mucosal hemorrhage. The only treatment for an acute episode is a transfusion of normal platelets. This patient has slightly decreased hemoglobin because of blood loss. Henoch Schonlein purpura is a self-limited autoimmune vasculitis that affects children and young adults, usually following an upper respiratory infection. Affected individuals develop purpuric rashes on the extensor surfaces of their arms, legs, and buttocks. They also have abdominal pain and hematuria from glomerulonephritis. Despite the tendency toward hemorrhage, the bleeding times and platelet count would be normal. Idiopathic thrombocytopenic purpura causes an increase in the bleeding time, but, as the name implies, platelet counts are decreased. There is bleeding from small vessels, especially of the skin, gastrointestinal tract, and genitourinary tract. Purpura and petechiae frequently develop. It is considered a self limited autoimmune disorder, typically affecting children after a recent viral infection. Thrombotic thrombocytopenic purpura, which is characterized by an increased bleeding time but a decreased platelet count, is a rare disorder of unknown etiology. It is thought to be initiated by endothelial injury, which releases certain procoagulant materials into the circulation, causing platelet aggregation. It causes purpura, fever, renal failure, microangiopathic hemolytic anemia, and microthrombi, generally in young women. In this disorder, platelet transfusion is actually contraindicated, as it can precipitate thrombosis. Von Willebrand disease causes increased bleeding times with normal platelet counts. It is the most common inherited bleeding disorder, caused by a defect in von Willebrand factor, which aids the binding of platelets to collagen. Even though the platelets them-selves are normal, binding is impaired, thus a platelet transfusion would not correct the problem. Cryoprecipitate, a plasma fraction rich in von Willebrand factor, would help in the case of von Willebrand disease but would not help with Bernard Soulier syndrome.
-
Question 78 of 100
78. Question
1 pointsFollowing flow volume curves were obtained by recording flow rate against volume during a forced expiration from maximum inspiration. The data were obtained from a normal subject and a patient with lung disease. The patient´s investigation result would likely include which data?
Correct
Incorrect
Explanation:
The patient data showing a decreased maximum flow rate and the total volume exhaled in addition to decreased total lung capacity (4 liters compared to 7 liters) is consistent with restrictive lung disease. Restrictive lung disease reduces not only total lung capacity and vital capacity, it also markedly reduces functional residual capacity (FRC) and, to a lesser extent, residual volume. In restrictive lung disease, both FEV and FVC are reduced, but typically the FEV1/FVC% is normal or increased. A decreased ratio is characteristic of obstructive lung disease. A patient with advanced restrictive disease is more likely to have reduced arterial oxygen and increased carbon dioxide due to low ventilation; if not, blood gases would be normal not reflecting higher than normal alveolar ventilation as in choice C. Anatomic dead space remains relatively constant at different lung inflations. The increased elastic recoil of the lung in a patient with restrictive lung disease decreases pulmonary compliance.
-
Question 79 of 100
79. Question
1 pointsA female presents with bilateral discharge of fluid from her nipples is suspected to have a hormone secreting pituitary tumor. Which hormone closest structural homology to the hormone likely responsible for her nipple discharge?
Correct
Incorrect
Explanation:
The hormone secreted by the pituitary that would cause nipple discharge (galactorrhea) is prolactin. Prolactin and growth hormone have very close structural homologies, and are considered to form the “prolactin-growth hormone family.” ACTH and melanocyte stimulating hormone (MSH) are in the “ACTH family.” FSH, LH, and TSH, have similarities, and form the “glycoprotein hormone family.”
-
Question 80 of 100
80. Question
1 pointsA 52 year old man has a 30 year history of smoking. For approximately 6 months per year for the preceding 5 years, he has had copious sputum production. This sputum production is most likely by which cells of bronchioles and small bronchi?
Correct
Incorrect
Explanation:
The patient has chronic bronchitis, which has been clinically seen and to be chronic cough with copious sputum production that has been present for at least 3 months in 5 consecutive years. Smoking and inhalation of other irritating substances including (smog) are usually associated. While the sputum production of early chronic bronchitis is the result of hypertrophy of submucosal glands in larger bronchi, in late and more severe cases, goblet cell metaplasia of bronchioles and small bronchi also contributes significantly to the mucus production. Other microscopic changes that occur include increased infiltration of small airways by alveolar macrophages and fibrosis of bronchiolar walls (that may even obliterate lumina). Smooth muscle hypertrophy can be seen in larger bronchi and is related to the increased effort needed to clear bronchi of copious amounts of mucus. However, it does not cause increased mucus production. Squamous cell metaplasia is seen as a reaction to irritation (often by smoking) in the larger bronchi. It is related to increased risk of squamous cell carcinoma in smokers, but not increased mucus production. Type I pneumocytes are the normal, flat lining cells of alveoli. Type II pneumocytes produce surfactant, not mucus. They can be increased as a reaction lo lung damage by processes such as pneumonia, which has an increased incidence in the setting of chronic mucus production with impaired clearance of bacteria.
-
Question 81 of 100
81. Question
1 pointsA child is diagnosed with idiopathic thrombocytopenic purpura. The mother is worried if her child will develop serious bleeding. When reassuring this mother, it might be helpful to inform her that the threshold for generalized bleeding is about how many platelets/mm3?
Correct
Incorrect
Explanation:
Normal platelet counts are in the range of 150,000to 450,000/mm3 it is useful to know that peripheral blood platelet counts less than about 15,000-20,000/mm3 are required before generalized bleeding usually occurs. Few physicians choose to actively treat patients who are not currently bleeding and whose platelet counts are greater than 100,000/mm3. Clearly, patients with counts less than 15,000/mm3 probably require active therapy, possibly with platelet transfusions. However, there is more disagreement about how to handle patients with counts in the 20,000-50,000/mm3 range, and you may encounter some physicians who will attempt to transfuse these patients. (However, this practice is no longer recommended by the American Blood Bank Association unless the patient is actively bleeding.) Idiopathic thrombocytopenic purpura is due to an autoimmune attack on the platelets, leading to their clearing by the spleen. Splenectomy may be helpful in refractory cases. A count of 5,000 platelets/mm3 is dangerously low, and platelet transfusion should have been considered earlier. Some physicians will give platelet transfusions for actively bleeding patients with counts of 50,000/mm3. Platelet counts of 100,000/mm3 do not usually require platelet transfusion, even when bleeding. Platelet counts of 150,000/mm3 do not usually require platelet transfusion, even when bleeding.
-
Question 82 of 100
82. Question
1 pointsA patient complains to the clinician that she feels lightheaded and has even fainted during defecation. This is likely an example of syncope due to which one of the following mechanisms?
Correct
Incorrect
Explanation:
Syncope has a broad differential diagnosis, since fainting can be produced by a wide variety of mechanisms. All of the mechanisms listed in the answers can produce syncope, but only the Valsalva mechanism (in which high intra-abdominal pressures trigger a reflex fall in cardiac output) is specifically associated with defecation. This mechanism can also produce fainting during weight-lifting and with the use of wind instruments. Anoxic “seizures” (nonepileptic) are fainting spells that occur because a patient holds his/her breath while experiencing severe pain or intense emotion. Hyperventilation, typically related to anxiety can also cause fainting. Patients who are hypovolemic because of medical reasons (e.g., hemorrhage, acute sodium or water loss, burns, Addison disease) tend to be hypotensive and may faint. Patients with cardiac arrhythmias are prone to fainting spells.
-
Question 83 of 100
83. Question
1 pointsA 41 year old female is concerned because her mother took DES for the prevention of spontaneous abortion. This woman should be told that her mother has an increased risk for developing which disease?
Correct
Incorrect
Explanation:
A 41 year old female is concerned because her mother took DES for the prevention of spontaneous abortion. This woman should be told that her mother has an increased risk for developing which disease?
-
Question 84 of 100
84. Question
1 pointsThe Mantoux reaction is an example of which type of hypersensitivity reaction?
Correct
Incorrect
Explanation:
The tuberculin skin test is an example of type IV hypersensitivity, or delayed type hypersensitivity (DTH) reaction. This reaction develops when primed Th l cells encounter their specific antigen. An inflammatory response evolves over 24-72 hours.
In the tuberculin skin test, the injected antigen is protein derived from M. tuberculosis. Th1 cells recognise peptide bound to MHC on APCs and are activated to secrete proinflammatory cytokines including IL 2, IFN-gamma, TNF, chemokines and GM-CSF. There is recruitment of inflammatory cells, predominantly macrophages, to the site of antigen deposition, with activation of phagocytes.
Some cytokines (TNF) as well as macrophage derived lytic enzymes cause local tissue destruction. CD8+ T-cells have also been implicated in DTH responses. The result is an indurated erythematous lesion at the site of injection which indicates previous exposure to TB. -
Question 85 of 100
85. Question
1 pointsA 37 year old woman is in her first pregnancy, which has been uneventful until the 34th week, when she develops swelling of feet and hands. An obstetric checkup shows that she also has hypertension and proteinuria. Investigations reveal an elevated aspartate aminotransferase (AST) and alanine aminotransferase (ALT), and slightly decreased platelets. What is the initial event in the pathogenesis of her condition?
Correct
Incorrect
Explanation:
This patient´s condition is a classic third-trimester complication referred to as toxemia of pregnancy or preeclampsia. Preeclampsia is defined as “sustained elevation of HP >140/90 after 20 weeks of pregnancy with proteinuria.” It occurs in 6% of all pregnancies but is more frequent in primiparas. Although the pathogenesis is still unclear, the first event appears to be placental ischemia, probably due to abnormalities in the trophoblast and alterations in the maturation of placental vessels. The trophoblast of invading placental vessels fails to acquire the characteristics of normal endothelial cells, with subsequent alterations in blood flow. Placental ischemia then triggers the release of thromboplastic substances, increases renin synthesis, and reduces prostaglandin E levels.
Chorioamnionitis (choice A) is an infection of chorioamnionic membranes due to bacteria that ascend through the vaginal canal. This is an important cause of spontaneous abortion in the second and third trimesters but plays no role in the pathogenesis of toxemia of pregnancy, The release of thromboplastic substances may cause disseminated intravascular coagulation (DIC) (choice B)
Increased renin and reduced prostaglandin E mediate increased sensitivity to angiotensin, leading to maternal hypertension (choice C)
Ischemic damage to maternal organs, including brain, liver, and kidneys (choice D) results from thrombotic occlusion of arterioles and capillaries as a consequence of DIC.
Overall, the clinical picture of toxemia is due to DIC mediated ischemic damage to brain (changes in mental status and convulsions), liver (elevated liver enzymes), and kidneys (proteinuria, leading to peripheral edema).A manifestation of toxemia is HFLLP syndrome which stands for hemolysis, elevated liver enzymes, and low platelets. -
Question 86 of 100
86. Question
1 pointsA 46 year old man living in Arizona has flu like symptoms that include fever, headache, and backache with cough. There is arthralgia of the knee and ankle joints. Erythema nodosum is present. Pneumonitis is detected on CXR. Organism that is most likely the cause of the patient´s symptoms is
Correct
Incorrect
Explanation:
While all of the answer choices may produce pulmonary symptoms, only Coccidiodes immitis produces erythema nodosum. The other findings of backache, flu like symptoms, and pneumonitis are all consistent with a diagnosis of coccidiomycosis. -
Question 87 of 100
87. Question
1 pointsA 19 year old sexually active female visits her physician complaining of increasing urinary frequency. She is diagnosed with a urinary tract infection. The most likely infectious cause is
Correct
Incorrect
Explanation:
Staphylococcus saprophyticus is responsible for one fifth of all urinary tract infections in women between the ages of 16 and 30. Although each of the others could potentially cause a urinary tract infection, Staphylococcus saprophyticus is the most obvious choice.
-
Question 88 of 100
88. Question
1 pointsAn 18 year old mentally retarded boy is brought because of recurrent epilepsy which is uncontrolled. Exam shows facial nevi and nodules in the retina. No other visible rashes on the body are present. His sister has similar symptoms. The most likely diagnosis is Correct
Incorrect
Explanation:
Tuberous sclerosis is an autosomal dominant neurocutaneous disorder which presents with seizures and adenoma sebaceum. Syringomyelia is a neurological disorder presenting without a rash and is secondary to a cavity within the central spinal cord. Neurofibromatosis is an autosomal dominant disorder. Café-au-lait spots are seen which are macular and greater than 1.5 cm in diameter, presenting mainly on the trunk and axilla and greater than six in number. Arnold Chiari malformation is secondary to a developmental displacement of the cerebrellar tonsils through the foramen magnum. Progressive cerebellar ataxia occurs. There is no rash. Meningococcal meningitis can be rapidly fatal if not treated early. The patient would appear toxic and have a rash on the lower extremities or trunk.
-
Question 89 of 100
89. Question
1 pointsA 46 year old female with rheumatoid arthritis develops swelling and tenderness near the angle of her right jaw. What other probable symptoms would also be present in her?
Correct
Incorrect
Explanation:
This patient´s symptoms are consistent with Sjögren´s syndrome, with parotitis (the swelling near her jaw) and keratoconjunctivitis sicca (dry eyes). Epistaxis, nuchal rigidity, painful pinnae, and bronchitis are not typically associated with this syndrome.
-
Question 90 of 100
90. Question
1 pointsA female aged 26 years with sclerodactyly, telangiectases, and Raynaud´s phenomenon presents with cramping diarrhea. Which one of the following is the most likely diagnosis in this patient?
Correct
Incorrect
Explanation:
The patient is likely to have scleroderma (progressive systemic sclerosis), which is often associated with a blind loop syndrome due to abnormal motility of the intestine. This constellation of symptoms is not characteristic of diverticulitis, appendicitis, gastroesophageal reflux, or Crohn´s disease.
-
Question 91 of 100
91. Question
1 pointsWhich one of the following does NOT promote infection with Group A Streptococci?
Correct
Incorrect
Explanation:
Streptococcal pyogenes is a normal resident of human mucous membranes and skin. Adherence of the bacterium to the mucosal epithelium is necessary for infection to occur. Fibronectin binding protein (protein F) contributes to adherence. Mutants lacking this protein are incapable of binding to epithelial cells. Streptococci may evade opsonophagocytosis by destroying or inactivating complement derived chemoattractants or opsonins. Approaching phagocytes are destroyed by streptolysin O, which is secreted in high concentrations in the tissues. Pyrogenic exotoxins and some M protein fragments have the unique ability to interact with certain VBETA regions of T cell receptors in the absence of classic antigen processing by antigen-presenting cells. As a consequence, there is a massive clonal proliferation of T lymphocytes. -
Question 92 of 100
92. Question
1 pointsAn archaeologist who has been excavating a very old Middle Eastern site develops a pustule on his hand. Formation of a black eschar surrounded by expanding brawny edema occurs after rupture of the pustule. The most likely cause of this condition is
Correct
Incorrect
Explanation:
Anthrax forms extremely stable spores and has, intact, been encountered in very old (i.e., deep) dirt in Israeli and Arabian excavation sites. The causative organism Bacillus anthracis is found in many animal species, and humans can acquire the organism either through contact with animals or from locally contaminated soil. The pustule described in the question stem is called a “malignant pustule” and may be accompanied by lymphadenopathy. Most cases remain localized, but death can occur as the result of complications such as bacteremia, meningitis, and pneumonia. Bacillus is a large, boxcar shaped rod that forms spores that look like bamboo shoots. Cigar shaped yeast describes the tissue form of Sporothrix schenckii, the agent of rose gardener´s disease. The lesion in such a case would be a mycetoma, draining yellow granular pus through multiple sinus tracts. A gram negative rod requiring cysteine describes Francisella tularensis, which causes tularemia. The organism can persist for weeks to months but does not form spores that could survive for hundreds or thousands of years. Also, most human cases occur in the endemic areas of the United States, in Arkansas and Missouri. A gram negative rod with bipolar staining describes yersinia pestis, which causes bubonic plague. This organism does not form stable spores that can survive for extended periods of time. The hallmark of bubonic plague is the formation of buboes, which are black, necrotic lymph nodes that form at the site of the vector fleabite. A large spirochete describes Borrelia burgdorferi, which causes Lyme disease. In the first stage of this disease, there is a localized, expanding erythematous rash rather than an eschar. Furthermore Lyme disease occurs in the United States, Europe, and Asia, corresponding to the distribution of Ixodid ticks, which spread the infection.
-
Question 93 of 100
93. Question
1 pointsA gram positive, catalase positive, coagulase positive coccus is isolated from a patient sample. The organism is found in small numbers and the culture plate also grows many other organisms. What is the most likely source of the specimen?
Correct
Incorrect
Explanation:
The organism described is Staphylococcus aureus, which is a common colonizer of the nasopharynx, where it may be part of the normal flora. At other sites, it is much more likely to be a true pathogen. Clues that an organism is a colonist in a particular setting include the presence of many other organisms in significant numbers, and relatively small numbers of acute inflammatory cells in smears made from the patient´s material. Blood and urine are normally sterile, and the isolation of S. aureus from these sites would probably indicate true infection. Stool contains large numbers of bacteria, but Enterococcus would be much more likely to be present as part of the normal flora than S. aureus. The vagina usually has a mixed flora, but S. aureus is not a normal colonist at this site.
-
Question 94 of 100
94. Question
1 pointsA child aged 1 year develops voluminous watery diarrhea and vomiting. She is brought to the pediatrician by her parents and evaluated, then sent home with instructions for the parents to give the child an electrolyte replacement solution. The most likely causative agent is best described by which one of the following?
Correct
Incorrect
Explanation:
Rotavirus is the major cause of diarrhea in infants and children under the age of 2. The virus replicates in the intestinal mucosa, producing a profuse, watery, non-bloody diarrhea, often coupled with nausea and vomiting. Transmission is by the fecal oral route. The virus belongs to the reovirus family which is the only family of double-stranded RNA viruses. They are naked capsid viruses have extremely segmented genomes and are icosahedral and double shelled. Double-stranded DNA, enveloped complex virus describes the poxvirus family. Double stranded DNA, enveloped icosahedral virus describes hepadnavirus and herpesvirus families. Double stranded DNA, naked, icosahedral virus describes papillomaviruses, polyomaviruses, and adenoviruses. Single stranded DNA, naked icosahedral virus describes the parvoviruses.
-
Question 95 of 100
95. Question
1 pointsA neonate´s family history is strong for birth defects. Chromosomal analysis of fetal cells does not demonstrate any abnormalities. Immediately at birth, cells are extracted from the umbilical cord and sent for flow cytometric analysis. Analysis shows a population of cells that are both CD4+ and CD8+. These cells are which one of the following?
Correct
Incorrect
Explanation:
CD4 and CD8 are both markers for T cells, and mature T cells usually express one or the other but not both. However some of the immature T cells in the thymus (that can “leak” into the baby´s blood in small numbers) coexpress these markers, before the cells have “committed” to being either helper T or cytotoxic T cells. In adults, significant numbers of cells coexpressing CD4 and CD8 usually implies the existence of a non-Hodgkin lymphoma with a cell population similar to immature T cells. Normally T-cell precursors entering the thymus are double positive for CD4 and CD8. During thymic selection, cells that recognize MHC class II become CD4-positive only and differentiate into helper T lymphocytes; whereas cells that recognize MHC class I become CD8 positive only and differentiate into cytotoxic T lymphocytes.
Helper T cells are CD4+ and CD8-, suppressor T cells are CD4- and CD8+. Immature B cells and plasma cells will be both CD4- and CD8-, since these are T-cell markers. -
Question 96 of 100
96. Question
1 pointsA 21 year old college student working part time in a pediatric clinic develops a viral exanthem with a rash. Her blood is tested for specific antibodies to varicella zoster. Which antibody class´s antivaricella immunoglobulin would indicate that she is immune to chickenpox?
Correct
Incorrect
Explanation:
The immunoglobulin that represents past exposure to an antigen is IgG. It is the predominant antibody in the secondary response and is an important defense against bacteria and viruses. In the secondary response, a much larger amount of IgG antibody is produced than IgM, and the levels of IgG tend to persist for a much longer time than in the primary response. IgG is an opsonic antibody: It binds to Fc receptors on phagocytic cells to enhance the speed of phagocytosis. An antibody with two epsilon chains describes IgE. It is produced during atopic allergies but would not be associated with immunity to a viral pathogen. An antibody with a valence of 4 describes IgA. The valence of an antibody is the number of combining sites per molecule, and since IgA is a dimer, it has a total of four combining sites. An antibody with a valence of 10 describes IgM. IgM is a pentameric molecule with a total often combining sites. The antibody with the shortest half life describes IgD. This serves as the membrane receptor on the surface of mature naive B lymphocytes along with IgM. It has no known biologic functions other than as a membrane receptor.
-
Question 97 of 100
97. Question
1 pointsA 15 year old girl has a sudden onset of watery diarrhea tinged with blood. She was previously healthy. Two weeks ago she started a treatment for acne vulgaris with topical benzoyl peroxide and an oral antibiotic with a similar mechanism of action to macrolides that is mainly used to treat anaerobic infections. Examination reveals a slightly distended abdomen that is diffusely tender. Her temperature is 38.1°C (100.5°F). What is the most likely diagnosis?
Correct
Incorrect
Explanation:
Pseudomembranous enterocolitis is caused by the toxins produced by Clostridium difficile. Even if it is difficult to determine the exact antibiotic this patient is currently taking the report of a recently started therapy should instantly make you think of antibiotic associated diarrhea or pseudomembranous colitis. The oral antibiotic this patient is currently taking is clindamycin, which is an antibiotic commonly used (in addition to tetracyclines) for the treatment of moderate to severe acne. As stated in the question, it has a similar mechanism of action and resistance as macrolides and is generally used to treat infections caused by anaerobic and some gram positive organisms. Patients develop fever and abdominal pain, with diarrhea often containing leukocytes and blood. Characteristic features of the disease include the formation of inflammatory, yellow tan pseudomembranes in the colon. The diagnosis is made by the C. difficile toxin assay, and the initial treatment of choice is with metronidazole (an alternative is vancomycin). Gastroenteritis is an acute condition, usually caused by a virus or bacterium that presents with vomiting and diarrhea. There is typically no blood in diarrhea associated with gastroenteritis, and most of these infections are self-limited. There is no association with oral antibiotic intake irritable bowel syndrome is predominantly a pain syndrome of unknown etiology in which there is increased frequency of the normal peristaltic and segmentation contractions of the bowel. It is a chronic condition that produces episodic diarrhea with periods of constipation. Salmonella infection is a common cause of food poisoning. Findings include nausea, vomiting, abdominal pain, and diarrhea often with blood.
Ulcerative colitis is a chronic condition that has features of bloody diarrhea, abdominal pain and weight loss. It is limited to the large bowel, and is exclusively a mucosal disease. Ulcerative colitis involves predominantly the rectum, has no skip lesions (mucosa is affected in its entire extension), and does not cause oral or perianal complications. -
Question 98 of 100
98. Question
1 pointsA 48 year old man has enlargement of the facial bone structure, including a thickened nose, prognathism, and frontal bossing. His fingers are sausage shaped. His FBS and BP are both at the upper limit of normal range, although his GHRH level is modestly suppressed. A mutation in one of his following best describes his condition?
Correct
Incorrect
Explanation:
This patient´s symptoms are typical for acromegaly, resulting from excess growth hormone (GH) effects in a post-pubertal adult. GH is secreted by anterior pituitary somatotroph cells in response to the hypothalamic secretion of growth hormone releasing hormone (GHRH). The modest suppression of GHRH in this patient is the result of excess negative feedback by GH secreted by the tumor. GHRH receptors are G-protein coupled receptors (GPCR) that activate G a, and transduce their signals by activating adenylate cyclase and increasing intracellular cAMP. A mutation preventing hydrolysis of GTP by Ga will cause excess receptor mediated signaling to the pituitary cells, resulting in proliferation and excess GH secretion. Options A and B are incorrect because a hepatic tumor might secrete IGF-1 but it would be in response to GH receptor stimulation. The growth effects of GH, including those seen with acromegaly, are mediated by IGF-1 and IGF-1 is secreted by the liver. But for a hepatic tumor to secrete IGF-1 in response to a mutation in a receptor signal transduction pathway, the mutation would need to be in the cytoplasmic tyrosine kinase transduction pathway of the GH receptor. Option C is incorrect because somatostatin inhibits GH release, it does not stimulate it. Option D is incorrect for the reasons discussed for the correct answer E.
-
Question 99 of 100
99. Question
1 pointsA male aged 27 years has recurrent episodes of intensely pruritic vesicles symmetrically distributed on his trunk. On electron microscopy, granular deposition of IgA and complement is noted at the dermoepidermal junction. Which underlying condition predisposes patients to the described skin disorder?
Correct
Incorrect
Explanation:
Patients with celiac sprue (nontropical sprue or gluten sensitive enteropathy) are prone to the development of dermatitis herpetiformis, the dermatologic diagnosis in this patient. This gastrointestinal malabsorption syndrome is caused by an allergic, immunologic, or toxic reaction to the gliadin component of gluten and has a genetic predisposition. There is reversal of symptoms of celiac sprue with a gluten free diet. Hint: Even if you were unaware of the association between celiac sprue and dermatitis herpetiformis, the mention of IgA and complement as an element in the skin pathology should have increased your suspicion that the associated condition was immunologically related. Lactase deficiency is the most common disaccharidase deficiency (lactase is absent from the brush border of the small intestine). It results in milk intolerance, with symptoms of bloating, diarrhea, and cramping following ingestion of dairy products. Tropical sprue is a disorder of malabsorption of unknown etiology that may be caused by enterotoxigenic Escherichia coli. There is a high incidence in the tropics, where it occasionally occurs in epidemics. Treatment is with tetracycline and folic acid. Ulcerative colitis is associated with a dermatologic condition called pyoderma gangrenosum. This disorder is characterized by blue red ulcerations that surround purulent necrotic bases. Whipple disease is a systemic disorder characterized by clumps of periodic acid-Schiff (PAS)-positive macrophages (full of and surrounded by small bacilli) in the lamina propria of intestines and in mesenteric lymph nodes. It also leads to malabsorption and responds to tetracycline. -
Question 100 of 100
100. Question
1 pointsA 55 year old man has higher than normal language output and frequent paraphasic errors. Neurologic evaluation demonstrates that his comprehension of auditory and visual language is severely disturbed. He also has an inability to repeat language. He is suffering from which type of aphasia?
Correct
Incorrect
Explanation:
The question stem describes Wernicke, or receptive, aphasia. It is caused by a lesion in Wernicke´s area, which is located in the posterior part of the superior temporal gyms of the language dominant hemisphere.
Broca (expressive) aphasia is a disorder primarily of language output. Speech is slow and effortful, phrase length is short, and the patient tends to use only common nouns, verbs, and, occasionally, adjectives. Comprehension is relatively intact. Repetition is generally disturbed comparable to the amount of spontaneous output. It is produced by a lesion of Broca´s area, which is in the posterior part of the inferior frontal gyrus of the language dominant hemisphere. Conduction aphasia is produced by a lesion in the arcuate fasciculus, which is in the posterior inferior part of the parietal lobe of the language dominant hemisphere. This effectively disconnects Broca´s area from Wernicke´s area, resulting in the inability to repeat. Global aphasia occurs with the destruction of Broca´s area, Wernicke´s area, and the arcuate fasciculus, resulting in a combination of both Broca and Wernicke aphasias. Mixed transcortical aphasia results from damage to much of both Broca´s and Wernicke´s areas, but leaves the arcuate fasciculus intact. The patient has little spontaneous language output or comprehension but is still able to repeat.